Đến nội dung

Hình ảnh

Một vài kỹ thuật tính toán với tổng $\sum\limits_{k=m}^n f(k)$

* * * * * 10 Bình chọn

  • Please log in to reply
Chủ đề này có 61 trả lời

#41
hxthanh

hxthanh

    Tín đồ $\sum$

  • Hiệp sỹ
  • 3921 Bài viết
Chủ đề này lại bị bỏ quên lâu quá!
Up lên nào

$\fbox{Bài 24}$
Tính
$S_n=\sum\limits_{k=1}^n \dfrac{2k^4+k^2}{4k^4+1}$

Các bạn vào ủng hộ nhé!

#42
hxthanh

hxthanh

    Tín đồ $\sum$

  • Hiệp sỹ
  • 3921 Bài viết
$\fbox{Bài 25}$

Tính $\quad S=\sum\limits_{k=0}^{2n} \dfrac{(-1)^k}{C_{2n}^k}$

#43
hxthanh

hxthanh

    Tín đồ $\sum$

  • Hiệp sỹ
  • 3921 Bài viết
Tiếp tục làm mới topic này :P

$\fbox{Bài 26}$

Tính: $\quad S=\sum\limits_{k=0}^n \left(\sum\limits_{j=0}^k C_n^j\right)^2$

#44
nthoangcute

nthoangcute

    Thiếu tá

  • Thành viên
  • 2003 Bài viết

$\fbox{Bài 24}$
Tính
$S_n=\sum\limits_{k=1}^n \dfrac{2k^4+k^2}{4k^4+1}$

Áp dụng cách tính của thầy Thanh ta được:
$${\frac {n \left( n+1 \right) \left( 1+2\,n \right) }{2\,{n}^{2}+2\,n+
1}}-{\frac { \left( n-1 \right) n \left( -1+2\,n \right) }{2\, \left(
n-1 \right) ^{2}+2\,n-1}}
=2\,{\frac {{n}^{2} \left( 2\,{n}^{2}+1 \right) }{ \left( 2\,{n}^{2}+2
\,n+1 \right) \left( 2\,{n}^{2}-2\,n+1 \right) }}$$
Từ đó ta tìm được
$$S_n=\sum\limits_{k=1}^n \dfrac{2k^4+k^2}{4k^4+1}=\frac{1}{2}\,{\frac {n \left( n+1 \right) \left( 1+2\,n \right) }{2\,{n}^{2}+
2\,n+1}}
$$
$

Bài viết đã được chỉnh sửa nội dung bởi nthoangcute: 30-07-2012 - 18:17

BÙI THẾ VIỆT - Chuyên gia Thủ Thuật CASIO

 

Facebook : facebook.com/viet.alexander.7


Youtube : youtube.com/nthoangcute


Gmail : [email protected]


SÐT : 0965734893


#45
nthoangcute

nthoangcute

    Thiếu tá

  • Thành viên
  • 2003 Bài viết

$\fbox{Bài 26}$
Tính: $\quad S=\sum\limits_{k=0}^n \left(\sum\limits_{j=0}^k C_n^j\right)^2$

Áp dụng cách tính thầy Thanh ta được:
$\quad S=\sum\limits_{k=0}^n \left(\sum\limits_{j=0}^k C_n^j\right)^2=(n+1)(2^n)^2$
Sau đó dễ dàng chứng minh bằng quy nạp được !

BÙI THẾ VIỆT - Chuyên gia Thủ Thuật CASIO

 

Facebook : facebook.com/viet.alexander.7


Youtube : youtube.com/nthoangcute


Gmail : [email protected]


SÐT : 0965734893


#46
nthoangcute

nthoangcute

    Thiếu tá

  • Thành viên
  • 2003 Bài viết

$\fbox{Bài 25}$
Tính $\quad S=\sum\limits_{k=0}^{2n} \dfrac{(-1)^k}{C_{2n}^k}$


Áp dụng cách tính thầy Thanh ta được:
$\quad S=\sum\limits_{k=0}^{2n} \dfrac{(-1)^k}{C_{2n}^k}=\frac{2n+1}{n+1}$
Sau đó dễ dàng chứng minh bằng quy nạp được !

BÙI THẾ VIỆT - Chuyên gia Thủ Thuật CASIO

 

Facebook : facebook.com/viet.alexander.7


Youtube : youtube.com/nthoangcute


Gmail : [email protected]


SÐT : 0965734893


#47
nthoangcute

nthoangcute

    Thiếu tá

  • Thành viên
  • 2003 Bài viết

$\fbox{Bài 19}$
Tính:
$S=\dfrac{3^3+1^3}{2^3-1^3}+\dfrac{5^3+2^3}{3^3-2^3}+\dfrac{7^3+3^3}{4^3-3^3}+...+\dfrac{4012^3+2006^3}{2007^3-2006^3}$

Áp dụng cách tính của thầy Thanh ta được:
Đặt: $S_{n}=\frac{n(3n+5)}{2}$
Suy ra $S_{n}-S_{n-1}=3n+1=\frac{(2n+1)^3+n^3}{(n+1)^3-n^3}$
Do đó, khi $n=2006$ thì ta được $S=\frac{n(3n+5)}{2}=6041069$

BÙI THẾ VIỆT - Chuyên gia Thủ Thuật CASIO

 

Facebook : facebook.com/viet.alexander.7


Youtube : youtube.com/nthoangcute


Gmail : [email protected]


SÐT : 0965734893


#48
nthoangcute

nthoangcute

    Thiếu tá

  • Thành viên
  • 2003 Bài viết

$\fbox{Bài 20}$
Dãy $\{S_n\}$ được định nghĩa bởi
$S_n=1+\dfrac{1}{1+2}+\dfrac{1}{1+2+3}+...+\dfrac{1}{1+2+...+n}\quad(n=1,2,...)$

Tìm số hữu tỉ $r$ nhỏ nhất sao cho: $S_n<r,\;\;\;\forall n\in\mathbb N^*$

Áp dụng cách tính của thầy Thanh ta được:
$S_{n}=2-\frac{2}{n+1}$
(Dễ dàng chứng minh theo quy nạp)
Suy ra $S_n<2$
Vậy $r=2$

BÙI THẾ VIỆT - Chuyên gia Thủ Thuật CASIO

 

Facebook : facebook.com/viet.alexander.7


Youtube : youtube.com/nthoangcute


Gmail : [email protected]


SÐT : 0965734893


#49
nthoangcute

nthoangcute

    Thiếu tá

  • Thành viên
  • 2003 Bài viết

$\fbox{Bài 23}$
Tính $ S_n=\sum\limits_{k = 1}^n\dfrac{k^2-k-1}{(k+1)!} $

Áp dụng cách tính của thầy Thanh ta được:
$ S_n=\sum\limits_{k = 1}^n\dfrac{k^2-k-1}{(k+1)!} =\frac{-n}{(n+1)!}$
(Dễ dàng chứng minh theo quy nạp)
Vậy $S_n =\frac{-n}{(n+1)!}$

BÙI THẾ VIỆT - Chuyên gia Thủ Thuật CASIO

 

Facebook : facebook.com/viet.alexander.7


Youtube : youtube.com/nthoangcute


Gmail : [email protected]


SÐT : 0965734893


#50
BoFaKe

BoFaKe

    Thiếu úy

  • Thành viên
  • 613 Bài viết
Sao toàn áp dụng công thức thầy Thanh thế này thì hiểu sao được :wacko: .
P/S đọc cái của thầy dễ tẩu hỏa quá :lol: ,phải nghiên cứu mới được.
~~~~~~~~~~~~~~Tiếc gì mà không click vào nút like mọi ngươì nhỉ ^0^~~~~~~~~~~~~~

#51
hxthanh

hxthanh

    Tín đồ $\sum$

  • Hiệp sỹ
  • 3921 Bài viết
$\fbox{Bài 27}$
Với các số nguyên dương $n>p>1$. Tính tổng:
$S=\sum\limits_{k=1}^n \left(2^k \left\lfloor\dfrac{k}{p}\right\rfloor\right)$

#52
hxthanh

hxthanh

    Tín đồ $\sum$

  • Hiệp sỹ
  • 3921 Bài viết

$\fbox{Bài 27}$
Với các số nguyên dương $n>p>1$. Tính tổng:
$S=\sum\limits_{k=1}^n \left(2^k \left\lfloor\dfrac{k}{p}\right\rfloor\right)$

Bài này nếu chỉ sử dụng các cách tính thông thường thì hơi khó khăn một chút, tuy nhiên nếu như sử dụng phương pháp sai phân từng phần thì lại rất đơn giản!

Ta đặt: $\Delta f(k)=2^k=(2^{k+1}-2^k)\Rightarrow f(k)=2^k$
$g(k)=\left\lfloor\dfrac{k}{p}\right\rfloor$
$\Rightarrow \Delta g(k)=\left\lfloor\dfrac{k+1}{p}\right\rfloor-\left\lfloor\dfrac{k}{p}\right\rfloor=\begin{cases}1\quad:\quad k+1\;\vdots\;p \\ 0\quad:\quad k+1\not{\vdots}\;p\end{cases}$

Áp dụng sai phân từng phần ta có:

$S=\left\lfloor\dfrac{k}{p}\right\rfloor 2^k\left|\begin{align*} {}^{n+1} \\ {}_{k=1}\end{align*}\right. - \sum_{k=1}^n \left(2^{k+1}\left(\left\lfloor\dfrac{k+1}{p}\right\rfloor-\left\lfloor\dfrac{k}{p}\right\rfloor\right)\right)$

$\Rightarrow S=\left\lfloor\dfrac{n+1}{p}\right\rfloor 2^{n+1} - \sum_{1\le mp-1\le n} 2^{mp}\quad\quad($với $k=mp-1)$
$\Rightarrow S=\left\lfloor\dfrac{n+1}{p}\right\rfloor 2^{n+1} - \sum_{m=1}^{\left\lfloor\frac{n+1}{p}\right\rfloor} 2^{mp}$
$\Rightarrow S=\left\lfloor\dfrac{n+1}{p}\right\rfloor 2^{n+1} -\dfrac{2^p\left(2^{^{p\left\lfloor\frac{n+1}{p}\right\rfloor}}-1\right)}{2^p-1}$

_________________________________
Mở rộng hơn một chút ta có kết quả sau:
Với $n>p>1\ne a>0$

$\sum_{k=1}^n\left(a^k\left\lfloor\dfrac{k}{p}\right\rfloor\right)=\dfrac{1}{a-1}\left(a^{n+1}\left\lfloor\dfrac{n+1}{p}\right\rfloor-\dfrac{a^p\left(a^{^{p\left\lfloor\frac{n+1}{p}\right\rfloor}}-1\right)}{a^p-1}\right)$

#53
batigoal

batigoal

    Hướng dẫn viên $\LaTeX$

  • Thành viên
  • 261 Bài viết
1. Cảm ơn anh Thanh rất nhiều về bài viết rất hay này.
2. Để bạn đọc hiểu rõ hơn và áp dụng rộng rãi hơn anh Thanh có thể đưa ra danh sách một số tài liệu tham khảo cho lĩnh vực này được không ạ.

#54
hxthanh

hxthanh

    Tín đồ $\sum$

  • Hiệp sỹ
  • 3921 Bài viết

1. Cảm ơn anh Thanh rất nhiều về bài viết rất hay này.
2. Để bạn đọc hiểu rõ hơn và áp dụng rộng rãi hơn anh Thanh có thể đưa ra danh sách một số tài liệu tham khảo cho lĩnh vực này được không ạ.

Cảm ơn Quân đã ủng hộ bài viết này!
Quả thực mình cũng đang tìm kiếm nguồn tài liệu để có thể bổ sung và phát triển cho chuyên đề này. Tiếc là đến nay vẫn chưa tìm được tài liệu nào phù hợp. Chuyên đề này mang tính chia sẻ kinh nghiệm cá nhân hơn là một tài liệu khoa học chính thống, vì vậy mong ước của mình là cùng với mọi người xây dựng và phát triển nó lên cho xứng tầm một serminar :)

#55
hxthanh

hxthanh

    Tín đồ $\sum$

  • Hiệp sỹ
  • 3921 Bài viết

MỘT SỐ PHÉP BIẾN ĐỔI VỚI TỔNG HAI CHIỀU

 

Tổng hai chiều, nghĩa là phép lấy tổng được thực hiện theo cả hai biến chạy, có thể độc lập hoặc phụ thuộc lẫn nhau.

 

$S=\sum_{\substack{j\in A\\ k\in B}} f(j,k)\quad(25)$

 

trong đó: $A$ là tập giá trị của $j$; còn $B$ là tập giá trị của $k$

 

Nếu $j, k$ độc lập tuyến tính với nhau thì $(25)$ là một tổng "kép" nghĩa là để tính $S$ ta phải tính lần lượt theo từng biến chạy (biến nào trước cũng được)

Còn nếu $j, k$ ràng buộc với nhau bởi một điều kiện nào đó thì sao?

Sau đây tôi sẽ trình bày một số dạng tổng phụ thuộc tuyến tính có thể đưa về dạng độc lập tuyến tính.

 

Đẳng thức 1:

Nếu $f(j,k)=f(k,j)$ nghĩa là hàm số cần lấy tổng có dạng đối xứng, khi đó ta có:

 

$S=\sum_{a\le j\le k\le b} f(j,k)=\dfrac{1}{2}\sum_{j=a}^b\sum_{k=a}^b f(j,k)+\dfrac{1}{2}\sum_{k=a}^b f(k,k)$

 

Ví dụ:

$\begin{align*}\sum_{1\le j\le k\le n} (j+k)&=\dfrac{1}{2}\sum_{j=1}^n\sum_{k=1}^n (j+k)+\dfrac{1}{2}\sum_{k=1}^n (k+k)\\&=\dfrac{1}{2}\sum_{j=1}^n\left(jn+\dfrac{n(n+1)}{2}\right)+\dfrac{n(n+1)}{2}\\&=\dfrac{n^2(n+1)}{4}+\dfrac{n^2(n+1)}{4}+\dfrac{n(n+1)}{2}\\&=\dfrac{n(n+1)^2}{2}\end{align*}$

 

Đẳng thức 2:

Nếu $f(j,k)=f(k,j)$ nghĩa là hàm số cần lấy tổng có dạng đối xứng, khi đó ta có:

 

$S=\sum_{a\le j < k\le b} f(j,k)=\dfrac{1}{2}\sum_{j=a}^b\sum_{k=a}^b f(j,k)-\dfrac{1}{2}\sum_{k=a}^b f(k,k)$

 

Ví dụ:

$\begin{align*}\sum_{1\le j< k\le n} (j+k)&=\dfrac{1}{2}\sum_{j=1}^n\sum_{k=1}^n (j+k)-\dfrac{1}{2}\sum_{k=1}^n (k+k)\\&=\dfrac{1}{2}\sum_{j=1}^n\left(jn+\dfrac{n(n+1)}{2}\right)-\dfrac{n(n+1)}{2}\\&=\dfrac{n^2(n+1)}{4}+\dfrac{n^2(n+1)}{4}-\dfrac{n(n+1)}{2}\\&=\dfrac{n(n+1)(n-1)}{2}\end{align*}$

 

Đẳng thức 3:

Khi việc tính tổng (hai chiều) phải thực hiện tuần tự trước sau theo từng biến chạy, ta có thể tìm cách thay đổi thứ tự lấy tổng sao cho phù hợp nhất ...

 

$\sum_{a\le j\le k\le b}f(j,k)=\sum_{j=a}^b\sum_{k=j}^b f(j,k)=\sum_{k=a}^b\sum_{j=a}^k f(j,k)$

 

Tổng ở giữa phải lấy theo $k$ trước rồi đến $j$ còn tổng bên phải thì ngược lại!

 

Ví dụ:

Tính tổng: $S=\sum_{j=1}^n\sum_{k=j}^n k2^j$

 

Nhận thấy rằng, việc tính theo $k$ trước sẽ làm cho tổng phải tính sau cùng khá là phức tạp. Do đó ta sẽ chuyển về tính theo $j$ trước:

Ta có:

$\begin{align*}S&=\sum_{j=1}^n\sum_{k=j}^n k2^j\\&=\sum_{1\le j\le k\le n}k2^j\\&=\sum_{k=1}^n\sum_{j=1}^k k2^j\\&=\sum_{k=1}^n k(2^{k+1}-2)\\&=-n(n+1)+\sum_{k=1}^n k\Delta(2^{k+1})\\&=-n(n+1)+(n+1)2^{n+2}-4-\sum_{k=1}^n 2^{k+2}\\&=(n-1)2^{n+2}+4-n-n^2\end{align*}$

 

Tất nhiên là theo cách tính ban đầu ta cũng được kết quả tương đương

Ta có:

$\begin{align*}S&=\sum_{j=1}^n\sum_{k=j}^n k2^j\\&=\sum_{j=1}^n\dfrac{(j+n)(n-j+1)}{2} 2^j\\&=\sum_{j=1}^n (n^2+n+j-j^2)2^{j-1}\\&=\sum_{j=1}^n(n^2+n+j-j^2)\Delta(2^{j-1})\\&=\left[n^2+n+(n+1)-(n+1)^2\right]2^n-(n^2+n+1-1^2)2^0-\sum_{j=1}^n (-2j)2^j\\&=-n^2-n+\sum_{j=1}^n j\Delta(2^{j+1})\\&=-n^2-n+(n+1)2^{n+2}-4-\sum_{j=1}^n 2^{j+2}\\&=-n^2-n-4+(n+1)2^{n+2}-2^{n+3}+8\\&=(n-1)2^{n+2}+4-n-n^2\end{align*}$

 

Đẳng thức 4:

Đôi khi ta gặp một tổng hai chiều nhưng hàm lấy tổng lại khuyết đi một biến...

 

$\sum_{a\le j\le k\le b}f(j)=\sum_{k=a}^b\sum_{j=a}^k f(j)=\sum_{k=a}^b (b-a+1-k)f(k)$

 

(to be continue...)



#56
nthoangcute

nthoangcute

    Thiếu tá

  • Thành viên
  • 2003 Bài viết

Không thấy topic này hoạt động gì cả :(
Đẩy lên 1 phát

$\fbox{Bài 23}$
Tính $ S_n=\sum\limits_{k = 1}^n\dfrac{k^2-k-1}{(k+1)!} $

 

 

Em thử áp dụng một đẳng thức gần giống SPTP, được gọi là Abel transformation, hoặc Abel's Lemma
 
$$G_n=\sum^n_{k=1} g_k$$
$$\sum^n_{k=m} f_k g_k=f_nG_n-f_m G_{m-1}-\sum^{n-1}_{k=m} G_k (f_{k+1}-f_k)$$
____________________________________
Ta có $$G_n=\sum^n_{k=1} (k^2-k-1)=\dfrac{1}{3} n(n-2)(n+2)$$
Do đó $S_n=\dfrac{\dfrac{1}{3} n(n-2)(n+2)}{(n+1)!}-\sum^{n-1}_{k=1} \frac{1}{3} k (k-2)( k+2) \left(\frac{1}{(k+2)!}-\frac{1}{(k+1)!}\right)\\=\dfrac{\dfrac{1}{3} n(n-2)(n+2)}{(n+1)!}+\frac{1}{3} \sum^{n-1}_{k=1} \frac{k-2}{(k-1)!}\\=\dfrac{\dfrac{1}{3} n(n-2)(n+2)}{(n+1)!}+\frac{1}{3} \sum^{n-1}_{k=1} \Delta \left[ -\frac{1}{(k-2)!}\right]\\=\dfrac{ n(n-2)(n+2)}{3(n+1)!}- \frac{1}{3 (n-2)!}\\=-\frac{n}{(n+1)!}$$
____________________________________
OK?

 

 


Bài viết đã được chỉnh sửa nội dung bởi nthoangcute: 26-04-2013 - 18:46

BÙI THẾ VIỆT - Chuyên gia Thủ Thuật CASIO

 

Facebook : facebook.com/viet.alexander.7


Youtube : youtube.com/nthoangcute


Gmail : [email protected]


SÐT : 0965734893


#57
zipienie

zipienie

    Thiếu úy

  • Thành viên
  • 533 Bài viết

Chủ đề này hay quá, mọi người tiếp tục đóng góp đi nào

 

Cho trước các số nguyên dương $k, m, n$ sao cho  $1 \leq  k \leq  m \leq  n$. Tính giá trị của tổng
\[ \sum^{n}_{i\mathop{=}0}\frac{1}{n+k+i}\cdot\frac{(m+n+i)!}{i!(n-i)!(m+i)!}\]

 

Chứng minh rằng với mọi số nguyên tố  $p$ ta có :
$$\sum_{k=1}^{p-1}\left \lfloor \frac{k^{3}}{p} \right \rfloor+\sum_{k=1}^{p^{2}-3p+2}\left \lfloor \sqrt[3]{kp} \right \rfloor=p^{2}-3p+2$$ 

Cho  $n$ là một số nguyên dương. Chứng minh rằng:
\[ \binom{n}{0}^{-1} + \binom{n}{1}^{-1} + \cdots + \binom{n}{n}^{-1} = \frac{n+1}{2^{n+1}} \left( \frac{2}{1} + \frac{2^2}{2} + \cdots + \frac{2^{n+1}}{n+1} \right).  \]


Luận văn, tài liệu tham khảo toán học : http://diendantoanho...ảo/#entry499457

Sách, Luận Văn, Tài liệu tham khảo https://www.facebook...TailieuLuanvan/

#58
hxthanh

hxthanh

    Tín đồ $\sum$

  • Hiệp sỹ
  • 3921 Bài viết

Chỉ có bài này là hay :)

 

Cho trước các số nguyên dương $k, m, n$ sao cho  $1 \leq  k \leq  m \leq  n$. Tính giá trị của tổng
\[ \sum^{n}_{i\mathop{=}0}\frac{1}{n+k+i}\cdot\frac{(m+n+i)!}{i!(n-i)!(m+i)!}\]

Nhưng đề đúng ra phải là:

$\displaystyle \sum_{i=0}^n\dfrac{(-1)^i}{n+k+i}\cdot\dfrac{(m+n+i)!}{i!(n-i)!(m+i)!}$

Nếu đề đúng như vậy thì giải bằng phương pháp SPTP cho nó độc :D

 

Ta có:

$\displaystyle S_{n,m,k}=\sum_{i=0}^n\dfrac{(-1)^i}{n+k+i}\cdot\dfrac{(m+n+i)!}{i!(n-i)!(m+i)!}=\sum_{i=0}^n\dfrac{(-1)^i(m+n+i)...(n+k+i+1). (n+k+i-1)!}{i!(n-i)!(m+i)...(k+i+1).(k+i)!}$

$\displaystyle\quad =\sum_{i=0}^n\dfrac{(m+n+i)^{\underline{m-k}}}{(m+i)^{\underline{m-k}}}\cdot \dfrac{(-1)^i{n\choose i}{n+k+i-1\choose n-1}}{n}$

 

(Chú ý ký hiệu $(x)^{\underline{n}}=x(x-1)...(x-n+1)$)

Bây giờ ta sẽ tìm sai phân

Ta có:

$\Delta\left[\dfrac{(m+n+i)^{\underline{m-k}}}{(m+i)^{\underline{m-k}}}\right]=\dfrac{(n+k+i+2)...(m+n+i+1)}{(k+i+2)...(m+i+1)}-\dfrac{(n+k+i+1)...(m+n+i)}{(k+i+1)...(m+i)}$

$\quad =n(k-m)\cdot\dfrac{(m+n+i)^{\underline{m-k-1}}}{(m+i+1)^{\underline{m-k+1}}}$

$\Delta\left[(-1)^{i-1}.{n-1\choose i-1}{n+k+i-1\choose n}\right]=\dfrac{n+k}{n}(-1)^i.{n\choose i} {n+k+i-1\choose n-1}$

 

Do đó (theo SPTP) ta có:

$\displaystyle S_{n,m,k}=\dfrac{1}{n+k}\sum_{i=0}^n \dfrac{(m+n+i)^{\underline{m-k}}}{(m+i)^{\underline{m-k}}}\Delta\left[(-1)^{i-1}.{n-1\choose i-1}{n+k+i-1\choose n}\right]$

$\displaystyle = \dfrac{1}{n+k}\left.\left[\dfrac{(m+n+i)^{\underline{m-k}}}{(m+i)^{\underline{m-k}}}\cdot (-1)^{i-1}.{n-1\choose i-1}{n+k+i-1\choose n}\right]\right|_{i=0}^{n+1}$

$\quad +\dfrac{n(m-k)}{n+k}\sum_{i=0}^n (-1)^{i}.{n-1\choose i}{n+k+i\choose n} \cdot\dfrac{(m+n+i)^{\underline{m-k-1}}}{(m+i+1)^{\underline{m-k+1}}}$

$\displaystyle =\dfrac{(m-k)}{n+k}\sum_{i=0}^{n-1} \dfrac{(-1)^i(m+n+i)!}{(n+k+i+1) i!(n-1-i)!(m+i+1)!}$

 

$\Rightarrow S_{n,m,k}=\dfrac{m-k}{n+k}S_{n-1,m+1,k+2}=\dfrac{m-k}{n+k}\cdot\dfrac{m-k-1}{n+k+1}S_{n-2,m+2,k+4}=...=$

 

$=\dfrac{(m-k)(m-k-1)...(m-k-n+1)}{(n+k)(n+k+1)...(2n+k-1)}S_{0,m+n,k+2n}$

 

Ở đó $S_{0,m+n,k+2n}=\dfrac{(-1)^0 (m+n+0+0)!}{(0+k+2n+0) 0!(0-0)!(m+n+0)!}=\dfrac{1}{2n+k}$

 

Vậy $\boxed{\displaystyle\sum_{i=0}^n\dfrac{(-1)^i}{n+k+i}\cdot\dfrac{(m+n+i)!}{i!(n-i)!(m+i)!}=\dfrac{(m-k)^{\underline{n}}}{(n+k)_{n+1}}}$

 

(Ký hiệu Pochhammer $(x)_n=x(x+1)...(x+n-1)$)

____________________________

 

NX: Với điều kiện đề bài là $1\le k\le m\le n$ thì

$0\le (m-k)\quad$ và $\quad(m-k-n+1)\le 0$ thế nên trong tích $(m-k)(m-k-1)...(m-k-n+1)$ tồn tại một thừa số là $0$ và tổng cần tính $=0$



#59
dark templar

dark templar

    Kael-Invoker

  • Hiệp sỹ
  • 3788 Bài viết

Cho  $n$ là một số nguyên dương. Chứng minh rằng:

\[ \binom{n}{0}^{-1} + \binom{n}{1}^{-1} + \cdots + \binom{n}{n}^{-1} = \frac{n+1}{2^{n+1}} \left( \frac{2}{1} + \frac{2^2}{2} + \cdots + \frac{2^{n+1}}{n+1} \right).  \]

Bài này tham khảo ở đây (#3).Trong đường link có cả bài tổng quát nữa,nhưng bài giải bằng tiếng Anh nên chú chịu khó đọc nhé !


"Do you still... believe in me ?" Sarah Kerrigan asked Jim Raynor - Starcraft II:Heart Of The Swarm.

#60
masterlovely

masterlovely

    Lính mới

  • Thành viên
  • 4 Bài viết

Tính tổng $S=\sum_{k=1}^{n}\frac{k+2}{k(k+1)2^{k}}$






0 người đang xem chủ đề

0 thành viên, 0 khách, 0 thành viên ẩn danh